Экспериментальный результат не может быть объяснен теорией для системы масс 2 Spring 1

У нас есть 2 пружины и 1 массовая система в 2D, как показано на рисунке.

Система

Вот моя краткая попытка решения:

Ф Икс "=" Ф 1 Икс + Ф 2 Икс "=" к ( Икс + л ) я ^ к ( Икс л ) я ^ "=" 2 к Икс я ^ Икс ¨ + 2 к м Икс "=" 0 ,
Ф у "=" Ф 1 у + Ф 2 у "=" 2 к у ȷ ^ у ¨ + 2 к м у "=" 0.
Общее решение для этих уравнений:
Икс ( т ) "=" А грех ( ю т ) + Б потому что ( ю т ) ,
у ( т ) "=" С грех ( ю т ) + Д потому что ( ю т ) ,
где ю "=" 2 к м . Оценивая начальные условия следующим образом;
Икс ( 0 ) "=" Икс 0 Икс ( 0 ) "=" А грех ( 0 ) + Б потому что ( 0 ) "=" Б "=" Икс 0 ,
у ( 0 ) "=" у 0 у ( 0 ) "=" С грех ( 0 ) + Д потому что ( 0 ) "=" Д "=" у 0 ,
Икс ˙ ( 0 ) "=" В 0 Икс "=" 0 Икс ˙ ( 0 ) "=" А ю потому что ( 0 ) Икс 0 ю грех ( 0 ) "=" А "=" 0 ,
у ˙ ( 0 ) "=" В 0 у "=" 0 у ˙ ( 0 ) "=" С ю потому что ( 0 ) у 0 ю грех ( 0 ) "=" С "=" 0 ,
Икс ( т ) "=" Икс 0 потому что ( ю т ) , у ( т ) "=" у 0 потому что ( ю т ) .
Я проверил это решение с помощью другого метода, приведенного здесь в первом ответе, и они непротиворечивы. Обратите внимание, что в последнем уравнении есть небольшая ошибка, она должна быть м вместо 2 м ; вы можете перепроверить здесь в первом ответе.

Я сделал рисунок этого раствора, и вот он:

введите описание изображения здесь

На факультете мы провели этот эксперимент, и результат выглядит примерно так (тоже сделано мной):

введите описание изображения здесь

Точки показывают положение массы. Единственная разница между этими двумя картинками — фазовый сдвиг. Чтобы получить экспериментальную цифру, я добавляю фазовый сдвиг у ( т ) и;

у ( т ) "=" у 0 потому что ( ю т + ф ) , ф "=" арктический ( у 0 / Икс 0 ) .

А еще вот что: Когда мы проводили этот эксперимент в лаборатории, инструктор сказал, что Икс ( т ) и у ( т ) должен иметь фазовый сдвиг π / 2 , по отношению друг к другу, что означает, если Икс ( т ) потому что ( ю т ) затем у грех ( ю т ) и наоборот. И это был реальный случай в лаборатории.

Мой вопрос в том, как я могу получить этот фазовый сдвиг из уравнений -легально-? Или есть какое-то объяснение?

Редактировать:

Это 50 с м × 50 с м Икс у горизонтальная плоскость, поэтому к системе не применяется перегрузка. м "=" 570 г р и к 60000 д у н с м . Остаточная длина пружин составляет л 0 "=" 13 с м . Для проведения эксперимента мы сначала растягиваем обе пружины и прикрепляем их к массе. Новое равновесие наступает, когда длина пружин составляет около 25 с м . Я думаю, что это довольно большое растяжение, но насколько я знаю, эластичность не нарушена.

Вот короткие кадры нормальных мод и небольших колебаний: https://www.youtube.com/watch?v=eyEpFeZO9W8 В лаборатории мы поставили этот эксперимент с гораздо большими амплитудами в обоих направлениях. Я предоставлю некоторые реальные фотографии и данные, как только смогу.

+1 за усилия и исследования. В вашем анализе нет гравитации. Были ли колебания ограничены горизонтальной плоскостью? Или пружины были настолько тугими, что гравитацией можно пренебречь?
Что находится на осях ваших графиков? Являются ли они y (t) против x (t)?
Начальные условия определяют, будут ли колебания для Икс и у оси будут синфазными или нет. Я думаю, что эксперимент и теория используют разные начальные условия.
Пожалуйста, не могли бы вы предоставить более подробную информацию о вашем эксперименте: какова длина покоя и длина равновесия каждой пружины? Каковы значения к и м ? Это у направление вертикальное, или Икс у плоскость горизонтальная? Как заставить систему колебаться? Каковы ваши реальные результаты? (Я предполагаю, что ваш последний график является иллюстрацией, а не фактическими данными.)
Это 50 с м × 50 с м Икс у горизонтальная плоскость, поэтому к системе не применяется перегрузка. м "=" 570 г р и к 60000 д у н с м . Остаточная длина пружин составляет л 0 "=" 13 с м . И да, я сделал графики в этом вопросе. Оси Икс у . Но реальные данные могу добавить завтра, после занятий.
Да, ваши фактические данные были бы очень полезны. Ты снял видео колебаний?
У меня есть хорошее видео нормальных режимов работы системы и действительно небольшая вибрация, постараюсь через минуту добавить это видео сюда. А актуальные данные я предоставлю завтра, также полное видео -если я могу добавить сюда видео-
@ Саба-Есть ли разница между первым растяжением пружин, после чего вы соединяете их с массой, и растяжением пружин с уже прикрепленной массой?
@Saba Можете ли вы предоставить данные своего эксперимента? например, таблицы x(t) и y(t). У вас также есть данные вашего измерения жесткости пружины?

Ответы (3)

Ваши уравнения движения неверны. Чтобы понять почему, рассмотрим случай на этой картинке:

введите описание изображения здесь

Какие Икс и у компоненты силы Ф действует на массу?

Если длина покоя пружины л 0 а его упругая постоянная к , сила Ф является

Ф "=" к р ^ ( л л 0 ) "=" к р ^ ( Икс 2 + у 2 л 0 )

где р ^ действует в направлении красной стрелки, т.е. р ^ "=" ( потому что θ , грех θ ) . Икс и у поэтому компоненты

Ф Икс "=" к потому что θ ( Икс 2 + у 2 л 0 ) Ф у "=" к грех θ ( Икс 2 + у 2 л 0 )

где

θ "=" арктический ( у Икс )

Если бы мы следовали методу, подобному вашему, мы бы получили

Ф Икс "=" к ( Икс л 0 ) Ф у "=" к у

что неправильно и соответствует случаю двух независимых пружин с одинаковыми константами, действующими на массу.

Возьмем случай с двумя одинаковыми пружинами:

введите описание изображения здесь

Основываясь на предыдущем анализе, вы можете легко увидеть, что

Ф 1 "=" к р ^ 1 ( Икс 2 + у 2 л 0 ) Ф 2 "=" к р ^ 2 ( ( Икс л ) 2 + у 2 л 0 )

где р ^ 1 "=" ( потому что θ 1 , грех θ 1 ) и р ^ 2 "=" ( потому что θ 2 , грех θ 2 ) . Отсюда следует, что Икс , у компоненты

Ф Икс "=" к потому что θ 1 ( Икс 2 + у 2 л 0 ) + к потому что θ 2 ( ( Икс л ) 2 + у 2 л 0 ) Ф у "=" к грех θ 1 ( Икс 2 + у 2 л 0 ) к грех θ 2 ( ( Икс л ) 2 + у 2 л 0 )

где

θ 1 "=" арктический ( у Икс ) θ 2 "=" арктический ( л Икс у )

Поэтому уравнения движения достаточно сложны для точного решения. Если вы умеете кодить, я бы предложил решить их каким-нибудь интегратором вроде Velocity Verlet .

Именно это и сделал Флорис в этом посте: physics.stackexchange.com/questions/231364/… . Для малых углов это приблизительно соответствует Fx=-2kx и Fy=-2T/L. Если вы запишете это, вы обнаружите, что пренебрегаемые члены более высокого порядка очень малы для высокой силы натяжения T даже при относительно больших смещениях, таких как x = y = L / 2.
@rickboender Не видел этого, спасибо за ссылку. Я могу расширить текущий ответ, включив в него обсуждение малых амплитуд. В любом случае вопрос ОП касается не только малых амплитуд (даже если на видео показан случай малых амплитуд).
@rickboender Было бы интересно посмотреть, может ли анализ малой амплитуды дать сдвиг фазы, по словам OP, мы должны получить, но я подозреваю, что нет ...
Я также не думаю, что фазовый сдвиг, который ожидает OP, может быть показан, частоты разные, поэтому на самом деле нет никакого фазового сдвига. Что касается малых перемещений, то обычно это единственный способ справиться с подобными проблемами. В нелинейных системах частота и демпфирование зависят от нагрузки, поэтому они постоянно меняются, и вам нужно точное измерительное оборудование для регистрации всего движения.

Расчет жесткости в у направление неверное. Жесткость в у -направление не зависит от жесткости пружины к , а только от силы натяжения Т в пружинах и длина пружин в состоянии покоя (прикрепленная к массе).

Жесткость в у - направление задается:

Ф "=" Т грех θ "=" Т   у л

Где θ угол между пружиной и осью X. Обратите внимание, что последний знак «=» действителен только для небольших смещений, как и для всего анализа. Если θ усиливается, эффект Т уменьшается с потому что θ , а влияние жесткости пружины увеличивается с грех θ .

Обычно фазового сдвига нет, потому что частоты в Икс и у направления разные. Они могут совпадать только для определенных значений к , длина л весны и Т . Если они совпадают, сдвиг можно определить с помощью начальных условий, как это сделали вы.

Если пружины натянуты Т в состоянии равновесия, а амплитуды малы, то восстанавливающие силы равны Ф Икс 2 к Икс и Ф у 2 Т л у , где л — длина пружины в растянутом состоянии, как описано в разделе Пояснение поперечных колебаний в системах с 1 массой и 2 пружинами . Если естественная длина пружин л 0 затем Т "=" к ( л л 0 ) так Ф у 2 к ( 1 л 0 л ) . Колебания в Икс и у направления ок. линейны и независимы, поэтому они являются простыми гармониками, но частоты отличаются соотношением ф у / ф Икс 1 л 0 л .

Эта разница в частоте означает, что разность фаз между Икс и у колебания постепенно усиливаются. Движение не похоже ни на один из ваших графиков, которые показывают постоянную разницу фаз. Вместо этого движение переходит от линейных колебаний, как на графике 1, к эллиптическим колебаниям на графике 2, которые становятся круговыми. Затем он снова становится эллиптическим, но на этот раз линия (= ось эллипса) отражается на оси y. После того, как снова станет линейным, направление колебаний меняется на противоположное, и цикл начинается снова. Это движение иллюстрируется анимацией в статье «Почему вибрация в моем проводе ведет себя так странно? » и можно увидеть в вашем видео также.

Используя предоставленные вами данные ( л 0 "=" 13 с м , л "=" 25 с м ) , затем 1 л 0 л 0,48 . Поэтому частоты должны быть в соотношении ф у / ф Икс 0,69 . Из 2-х прогонов в 1-й половине вашего видео примерно за 8 секунд 11 циклов Икс колебаний и 7 циклов у колебание, поэтому ф у / ф Икс "=" 7 / 11 0,64 , что достаточно близко к предсказанию.

Однако Икс и у колебания не кажутся независимыми друг от друга. Во 2-х работает во 2-й половине видео, в котором Икс и у движения происходят одновременно, отношение ф у / ф Икс составляет ок. 8 / 9 вместо 7 / 11 когда эти движения раздельны. Разница в частоте значительно меньше, и каждый сдвинулся навстречу другому. Этому есть две причины: (i) малоамплитудное приближение не выполняется, поэтому Ф Икс , Ф у каждый зависит от Икс и у - т.е. они связаны; (ii) энергия также передается через трение или гистерезис. (Для примера фрикционной связи двух независимых движений см. Вращательная физика игральной карты ).

Не очевидно, как π / 2 может возникнуть разность фаз, предложенная вашим учителем. Если бы одно колебание приводило в действие другое, оно опережало бы π / 2 . Это могло бы произойти, если бы были две связанные массы, одна из которых была бы намного тяжелее другой. Но здесь массы те же.

Из видео, ф Икс 11 / 8 1,4 ЧАС г . Судя по вашим измерениям, если предположить, что к относится к одной из двух пружин, то ф Икс 1 2 π 2 к м "=" 1 2 π 2 × 60 , 000 570 2.3 ЧАС г . Возможно, ваша стоимость к не является правильным?


Уравнение, которое вы вывели для Ф у применяется только тогда, когда л л 0 . Затем ф у / ф Икс 1 поэтому разность фаз остается прибл. постоянный. Если вы запускаете систему из состояния покоя, разность фаз равна нулю (график 1), потому что обе Икс и у начните с максимального смещения, чтобы они имели одинаковую фазу. Амплитуды не обязательно должны быть одинаковыми, потому что частота не зависит от амплитуды. Чтобы иметь постоянную разность фаз, как на графике 2, вы можете переместить массу в Икс или у направлении, когда вы отпустите его.

Если амплитуда колебаний становится «большой», то Икс и у колебания становятся нелинейными и связанными. Если натяжение пружин в состоянии равновесия очень мало или отсутствует, то поперечные колебания даже при малых амплитудах будут нелинейными, а восстанавливающая сила будет пропорциональна у 3 .